Saturday, March 09, 2019

Motte and Bailey, Particle Physics Style

“Motte and bailey” is a rhetorical maneuver in which someone switches between an argument that does not support their conclusion but is easy to defend (the “motte”), and an argument that supports their conclusion but is hard to defend (the “bailey”). The purpose of this switch is to trick the listener into believing that the easy-to-defend argument suffices to support the conclusion.

This rhetorical trick is omnipresent in arguments that particle physicists currently make for building the next larger collider.

There are good arguments to build a larger collider, but those don’t justify the investment. These arguments are measuring the properties of known particles to higher precision and keeping particle physicists occupied. Also, we could just look and see if we find something new. That’s the motte.

Then there is an argument which would justify the investment, but this is not based on sound reasoning. This argument is that a next larger collider would lead to progress in the foundations of physics, for example by finding new symmetries or solving the riddle of dark matter. This argument is indefensible because there is no reason to think the next larger collider would help answering those questions. That’s the bailey.

This maneuver is particularly amusing if you both have people who make the indefensible argument and others who insist no one makes it. In a recent interview with the CERN courier, for example, Nima Arkani-Hamed says:
“Nobody who is making the case for future colliders is invoking, as a driving motivation, supersymmetry, extra dimensions…”
While his colleague, Lisa Randall, has defended the investment into the next larger collider by arguing:
“New dimensions or underlying structures might exist, but we won’t know unless we explore.”
I don’t think that particle physicists are consciously aware of what they are doing. Really, I get the impression they just throw around whatever arguments come to their mind and hope the other side doesn’t have a response. Most unfortunately, this tactic often works, just because there are few people competent enough to understand particle physicists’ arguments and also willing to point out when they go wrong.

For this reason I want to give you an explicit example for how motte and bailey is employed by particle physicists to make their case. I do this in the hope that it will help others notice when they encounter this flawed argument.

The example I will use is a recent interview I did for a podcast with the Guardian. The narrator is Ian Sample. Also on the show is particle physicist Brian Foster. I don’t personally know Foster and never spoke with him before. You can listen to the whole thing here, but I have transcribed the relevant parts below. (Please let me know in case I misheard something.)

At around 10:30 min the following exchange takes place.

Ian: “Are there particular things that physicists would like to look for, actual sort-of targets like the Higgs, that could be named like the Higgs?”

Brian: “The Higgs is really, I think, at the moment the thing that we are particularly interested in because it is the new particle on the block. And we know very little about it so far. And that will give us hopefully clues as to where to look for new phenomena beyond the standard model. Because the thing is that we know there must be physics beyond the standard model. If for no other reason than, as you mention, there’s very strong evidence that there is dark matter in the universe and that dark matter must be made of particles of some sort we have no candidate for those particles at the moment.”

I then explain that this argument does not work because there is no reason to think the next larger collider would find dark matter particles, that, in fact, we are not even sure dark matter is made of particles.

After some more talk about the various proposals for new colliders that are currently on the table, the discussion returns to the question what justifies the investment. At about 24:06 you can hear:

Ian: “Sabine, you’ve had a fair bit of flak for some of your criticisms for the FCC, haven’t you, from within the community?”

Sabine: “Sure, true, but I did expect it. Fact is, we have no reason to think that a next larger particle collider will actually tell us anything new about the fundamental laws of nature. There’s certainly some constants that you can always measure better, you can always say, well, I want to measure more precisely what the Higgs is doing, or how that particle decays, and so on and so forth. But if you want to make progress in our understanding of the foundations of physics that’s just not currently a promising thing to invest in. And I don’t think that’s so terribly controversial, but a lot of particle physicists clearly did not like me saying this publicly.”

Brian: “I beg to differ, I think it is very controversial, and I think it’s wrong, as I’ve tried to say several times. I mean the way in which you can make progress in particle physics is by making these precision measurements. You know very well that quantum mechanics is such that if you can make very high precision measurements that can tell you a lot of things about much higher energies than what you can reach in the laboratory. So that’s the purpose of doing very high precision physics at the LHC, it’s not like stamp collecting. You are trying to make measurements which will be sufficiently precise that they will give you a very strong indication of where there will be new physics at high energies.”

(Only tangentially relevant, but note that I was talking about the foundations of physics, whereas Brian’s reply is about progress in particle physics in particular.)

Sabine: “I totally agree with that. The more precisely you measure, the more sensitive you are to the high energy contributions. But still there is no good reason right now to think that there is anything to find, is what I’m saying.”

Brian: “But that’s not true. I mean, it’s quite clear, as you said yourself, that the standard model is incomplete. Therefore, if we can measure the absolutely outstanding particle in the standard model, the Higgs boson, which is completely unique, to very high precision, then the chances are very strong that we will find some indication for what this physics beyond the standard model is.”

Sabine: “So exactly what physics beyond the standard model are you referring to there?

Brian: “I have no idea. That’s why I want to do the measurement.”

I then explain why there is no reason to think that the next larger collider will find evidence of new physical effects. I do this by pointing out that the only reliable indications we have for new physics merely tell us something new has to appear at latest at energies that are still about a billion times higher than what even the next larger collider could reach.

At this point Brian stops claiming the chances are “very strong” that a bigger machine would find something new, and switches to the just-look-argument:

Brian: “Look, it’s a grave mistake to be too strongly lead by theoretical models [...]”

The just-look-argument is of course well and fine. But, as I have pointed out many times before, the same just-look-argument can be made for any other new experiment in the foundations of physics. It therefore does not explain why a larger particle collider in particular is a good investment. Indeed, the opposite is the case: There are less costly experiments for which we have good reasons, such as measuring more precisely the properties of dark matter or probing the weak field regime of quantum gravity.

When I debunk the just-look-argument, a lot of particle physicists then bring up the no-zero-sum-argument. I just did another podcast a few days ago where the no-zero-sum-argument played a big role and if that appears online, I’ll comment on that in more detail.

The real tragedy is that there is absolutely no learning curve in this exchange. Doesn’t matter how often I point out that particle physicists’ arguments don’t hold water, they’ll still repeat them.

(Completely irrelevant aside: This is the first time I have heard a recording made in my basement studio next to other recordings. I am pleased to note all the effort I put into getting good sound quality paid off.)

108 comments:

  1. Hi Sabine,

    since you quoted Nima, taking a small extract of one of his answer, I think it would be good for your readers to report the full answer, because it raises other interesting points:

    "Nobody who is making the case for future colliders is invoking, as a driving motivation, supersymmetry, extra dimensions or any of the other ideas that have been developed over the past 40 years for physics beyond the Standard Model. Certainly many of the versions of these ideas, which were popular in the 1980s and 1990s, are either dead or on life support given the LHC data, but others proposed in the early 2000s are alive and well. The fact that the LHC has ruled out some of the most popular pictures is a fantastic gift to us as theorists. It shows that understanding the origin of the Higgs mass must involve an even larger paradigm change than many had previously imagined. Ironically, had the LHC discovered supersymmetric particles, the case for the next circular collider would be somewhat weaker than it is now, because that would (indirectly) support a picture of a desert between the electroweak and Planck scales. In this picture of the world, most people wanted a linear electron–positron collider to measure the superpartner couplings in detail. It’s a picture people very much loved in the 1990s, and a picture that appears to be wrong. Fine. But when theorists are more confused, it’s the time for more, not less experiments."

    I take this just as an example (there are many other interesting points raised in that interview that support building a collider). I agree with you, it is maybe true that some theorists jump from one argument to the other, but I think some of them make some points very clearly, like the ones above. Unfortunately, my impression is that, whenever this happens, you decide to ignore those arguments and only consider those that you can easily attack, in order to support your point of view.

    Sorry, this is my opinion of course, but at the end of the day, I kind of have the feeling that your way of arguing is no different than that of the particle physicists that you strongly criticise.

    Lorenzo

    ReplyDelete
    Replies
    1. Lorenzo,

      It is not correct that I ignore "other interesting points".

      I have address "other points" countless of times, in my blogposts, in comments on my blogposts, on facebook, on twitter, in interviews, and by email.

      The reason I have now decided to focus on one point at a time in my blogposts is that I hope in this way I will get across my arguments clearer.

      You want me to address that quote, fine, but stop accusing me that I am supposedly not addressing criticism, when the truth is that I have patiently answered to endless repetitions of the always same arguments for months.

      First let me note that the "fantastic gift" that the LHC has given particle physicists is to demonstrate without doubt that naturalness-based predictions do not work. It does not follow from this that there are any new particles to find at even higher energies. The "paradigm shift" that is necessary is that they realize these arguments do not work, not to insist that there is something funny about the mass of the Higgs. That's the opposite of learning from failure.

      Claiming that finding supersymmetric particles at the LHC would have made the case for the next larger collider weaker makes no sense. If the LHC had found at least one supersymmetric particles, then we would know now that there must be something more beyond the standard model. This would have justified building a larger collider. Alas, this justification never appeared.

      The argument that "when theorists are more confused, it’s the time for more, not less experiments" is correct of course, but suffers from the same problem as the "just look" argument, in that it does not explain why a larger collider is the best investment of money.

      Indeed, as I have said many times, there are good reasons why a next larger collider is *not* a good investment. If theorists are confused, what we need is evidence of new phenomena, not null results. When it comes to BSM searches, we have seen only null results. These null results are the reason theory development is stuck.

      What we need are experiments that will deliver (better) evidence of new phenomena. A next larger collider brings a high risk that all we will see in the next 30 years is further null results. It is not a good investment. If experiments costs tens of billions of dollars, we have to be careful when deciding what experiments to do. Particle physicists are not careful with their arguments.

      Delete
  2. Keep hurling those rocks at the bailey and the motte. :)

    ReplyDelete
  3. This seems a little uncharitable. The point of motte and bailey is that the _same_ person switches back and forth between a conclusion everybody agrees with, and a much more far-reaching conclusion that fewer do, dishonestly.

    You gave an example of two _different_ people (Nima and Randall) having different opinions. That's not motte and bailey, that's just people being different. It makes sense for Randall to be more excited about extra dimensions and Nima to be less excited, because Nima's idea (large extra dimensions) was strongly constrained by the LHC, while Randall's (Randall-Sundrum warped extra dimensions) wasn't as much. I know that it's annoying seeing different people say contradictory things, but if you want to understand how people are actually making the funding case (instead of what spectators, including particle phenomenologists, are saying about it), you need to read the actual funding proposals written at CERN. As Nima says, these proposals do not focus on stuff like SUSY or extra dimensions at all.

    Similarly, I think Brian is taking a perfectly self-consistent position here and you're misrepresenting it. His point is that a precision measurement of the Higgs can tell us a lot, independent of specific models or theoretical expectations, because many things can affect the Higgs indirectly. You bring up the "energy desert" theoretical argument that there should exist no new physics at all until a much higher scale. And he points out that this is just another one of those theoretical speculations with no direct evidence behind it. (Indeed, if the LHC told us anything about new physics, it's that the energy desert argument isn't right! That argument only came to be accepted because of SUSY and GUTs.)

    I'm not saying there aren't any dishonest people out there, but dishonesty isn't the same thing as different people having different views in casual conversation. CERN's official description of the motivation for the FCC (e.g. see here: https://cds.cern.ch/record/2651294/files/CERN-ACC-2018-0056.pdf) is the real thing, which is presumably what funding agencies will actually refer to. It devotes 1 page out of 150 to SUSY and one sentence to extra dimensions.

    ReplyDelete
    Replies
    1. Kevin,

      The example I have discussed is the one of Brian Foster. That's one person.

      Delete
    2. Kevin,

      I did comment explicitly on the short versions of the proposal, see here.

      As I have said many times, I did read parts of but not the entire full report. This report is all well and fine in explaining what we would get for $20 billion. It does nothing to explain why that is the best way to invest such an amount of money to make progress in the foundations of physics.

      So far, not a single particle physicist has addressed that point, which is the only relevant point. Why invest $20 billion in a larger particle collider and not in other, less costly experiments, that have better motiviations?

      Delete
  4. "There are good arguments to build a larger collider, but those don’t justify the investment. These arguments are measuring the properties of known particles to higher precision and keeping particle physicists occupied. Also, we could just look and see if we find something new. That’s the motte."

    And why do first measurements of an unknown sector of physics not justify the investment? I think this kind of reasoning just shows that you do not understand what experimental particle physics is about.

    "The just-look-argument is of course well and fine. But, as I have pointed out many times before, the same just-look-argument can be made for any other new experiment in the foundations of physics. It therefore does not explain why a larger particle collider in particular is a good investment. Indeed, the opposite is the case: There are less costly experiments for which we have good reasons, such as measuring more precisely the properties of dark matter or probing the weak field regime of quantum gravity."

    This is simply factually wrong and repeating it doesn't make it any better an argument. There are no other experiments that can probe anything close to what a future collider would be measuring, you can't study the Higgs sector through any of the less costly experiments you talk about. Saying that those other experiments are more promising is just a judgement call on your part that has absolutely no grounding on any evidence other than your gut feeling of where new physics should show up. Maybe you are right, maybe you are wrong, time will tell, unless we don't look of course.

    Also, believe it or not, Nima Arkani-Hamed and Lisa Randall are two distinct people that might have different thoughts on the subject of future colliders. I don't see how their statements are in contradiction either, I don't see Lisa claiming that extra dimensions are the driving motivation for the FCC - although clearly she thinks it's a more important factor than Nima, who seems to have a more nuanced view point on this (and I would side with Nima here).

    ReplyDelete
    Replies
    1. Fx,

      "why do first measurements of an unknown sector of physics not justify the investment?"

      As I have said many times, the reason that a next larger collider is currently not a good investment is that futher null results will not help us solve any of the problems we currently have in the foundations of physics.

      It is irrelevant, btw, whether you personally agree on whether or not measuring the Higgs serves as sufficient justification, the point is that that's a tough sell to the public, and I am sure you full well know that (as do all particle physicists).

      "This is simply factually wrong and repeating it doesn't make it any better an argument. There are no other experiments that can probe anything close to what a future collider would be measuring,...

      There is nothing factually wrong about my statement. Better measurements of dark matter are better motivated because in that case we already know that there is something new to find. Probing quantum gravity is better motivated because it's based on an argument from consistency. This is not a gut feeling, this is based on (a) experimental evidence and (b) sound mathematics. There is no similar argument for a next larger collider that is remotely as good.

      Your claim that what I say is wrong because you "cannot study the Higgs sector" this way makes no sense. I did not claim that measuring the Higgs sector is what those other experiments would do to begin with.

      Delete
  5. "It is irrelevant, btw, whether you personally agree on whether or not measuring the Higgs serves as sufficient justification, the point is that that's a tough sell to the public, and I am sure you full well know that (as do all particle physicists)."

    It is not the public that needs to be sold on the justification of measurements of the Higgs, but the scientific community. I do not really care what the wider public thinks, although obviously it would be nice if they are excited about particle physics.

    We do not know whether a larger collider will yield null results or find deviations. If it finds null results, we will nevertheless understand the Higgs sector thoroughly, which is an extremely valuable addition to our understanding of the universe, and in that null-result scenario perhaps the last piece we will ever be able to actually measure in our lifetime. If the proposal ends up not being funded, fine, it would be a pity but resources are limited and such is life, but I think it is ludicrous to say out of the gate that such an experiment is not a good investment.

    "Better measurements of dark matter are better motivated because in that case we already know that there is something new to find. Probing quantum gravity is better motivated because it's based on an argument from consistency. This is not a gut feeling, this is based on (a) experimental evidence and (b) sound mathematics. There is no similar argument for a next larger collider that is remotely as good."

    Dark matter searches have come up empty so far, and there is no particularly strong reason to expect a discovery any time soon (although I hope we do obviously). Quantum gravity is a very difficult thing to probe and while I am 100% behind the idea of funding these smaller scale table top experiments, they are for the most part long shot ideas. There is absolutely no reason to expect that increasing funding in that area will lead to a massive discovery. That is not to say these areas are not worth pursuing, of course they are, but it is absolutely your gut feeling that they are "more promising" as an avenue for the next big discovery.

    ReplyDelete
    Replies
    1. Fx,

      You write

      "It is not the public that needs to be sold on the justification of measurements of the Higgs, but the scientific community."

      First, it is a grave mistake of you to think that the people who actually pay for your experiment do not need to be explained whether their money is being put to good use. (And an even graver mistake to say that in public, btw.)

      Second, you are not "convincing" anyone besides particle physicists who need no convincing. I find it mildly amusing, that you seem to think just saying you want to measure the Higgs some more is going to suffice to get $20 billion dollar, but I have no particular objection to you trying that.

      "We do not know whether a larger collider will yield null results or find deviations"

      I did not say that that we know that. Stop pretending I said things I did not say.

      "it is ludicrous to say out of the gate that such an experiment is not a good investment."

      I do not say this "out of the gate", I say this after having spent several years thinking about the question why there has not been progress in the foundations of physics for 40 years. I literally wrote a whole book about it.

      Since you probably think the question why there has been no progress in the foundatations of physics for 40 years is not interesting, and therefore reading what I said about the topic is unnecessary, let me give you the brief summary: Null-results do not aid theory-development very much. They tell you which direction not to go rather than telling you where to go. To make progress, we need evidence of new phenomena, not more null results. "Just look" carries a high risk of further null results. "Just look" carries a high risk of leaving us stuck for even longer.

      Historically, breakthroughs have come from resolving inconsistencies between theory and experiment or from resolving internal theoretical inconsistencies. Therefore, the areas where such problems exist are the ones we should focus on to make progress.

      "Dark matter searches have come up empty so far..."

      You are misunderstanding what I am talking about. I am NOT talking about building more detectors for particles that we do not know exist. We have seen enough null-results this way. I am saying study closer the already known discrepancy between theory and experiment.

      "There is absolutely no reason to expect that increasing funding in that area will lead to a massive discovery."

      In quantum gravity, the case of a breakthrough discovery is considerably more plausible, plus the experiments are less costly. Ie, less risk and higher benefit than a next larger collider. Therefore, we should finance these experiments first.

      Delete
    2. Given that we probably won't see anything BSM with a 20 billion plus collider, and that we won't see dark matter with a 1 billion dark matter experiment, doesn't it make sense to build the dark matter experiment?

      Delete
  6. "First, it is a grave mistake of you to think that the people who actually pay for your experiment do not need to be explained whether their money is being put to good use. (And an even graver mistake to say that in public, btw.)"

    Oh come on, I am not saying we should not explain to the public what we are doing with their money, I am saying they don't have the background to judge which experiment are most interesting and funding should rely on expert panels.

    "Since you probably think the question why there has been no progress in the foundations of physics for 40 years is not interesting, and therefore reading what I said about the topic is unnecessary, let me give you the brief summary: Null-results do not aid theory-development very much. "

    I do think it is important to ask ourselves if we are taking the right approach, but my conclusion differs from yours. First of all I think it is not really true to say there has been no progress. I also disagree that what is needed is to throw out everything we have done so far, call it a day and do something else. Physics is hard, and sometimes progress has to be hard-earned. As the title of Nima's interview summarizes well, progress in collider physics will be slow and if you don't have the patience for it you should work on other things, but that does not make it an invalid approach. I also think you are deluding yourself in thinking that progress will be fast and easy by looking at dark matter or quantum gravity if we were to just invest more in those areas.

    "Historically, breakthroughs have come from resolving inconsistencies between theory and experiment or from resolving internal theoretical inconsistencies. Therefore, the areas where such problems exist are the ones we should focus on to make progress."

    You keep saying that and we already had a long exchange on this on one of your previous post where I commented on. I will repeat what I already said there, which is that this is just not true, breakthroughs can come in a variety of forms, and there has been a number of discoveries that did not come from resolving previously known inconsistencies, like the Rutherford experiment, the CMB discovery, etc.

    "You are misunderstanding what I am talking about. I am NOT talking about building more detectors for particles that we do not know exist. We have seen enough null-results this way. I am saying study closer the already known discrepancy between theory and experiment. "

    I did not misunderstand you, but astrophysical searches have also come up empty so far, and while we know there is something to be found there is no guarantee that we will find it or that it can even be found from the experiments we are designing. Perhaps we will find nothing for 50 years here too. You can't guarantee non-null results, be it at the FCC or in DM experiment or in QG probes.

    "In quantum gravity, the case of a breakthrough discovery is considerably more plausible, plus the experiments are less costly. Ie, less risk and higher benefit than a next larger collider. Therefore, we should finance these experiments first."

    Please explain to me how you quantify that these discoveries are "more plausible". To me this is just wishful thinking. The fact that these experiments are less costly does not make them more likely to find something.
    Furthermore, science funding does not work as a priority queue where only the "most promising" experiment gets funding and we wait until we see the results to fund something else. It is also a game of maximizing the cross section of things we look at and taking several orthogonal approaches to tackle different problems in different ways.

    ReplyDelete
    Replies
    1. Fx,

      "Oh come on, I am not saying we should not explain to the public what we are doing with their money,"

      Well, you previously wrote "I do not really care what the wider public thinks". But good, then, if you do care after all, how about you explain why that collider is a good way to invest the money?

      " First of all I think it is not really true to say there has been no progress."

      I have been exceedingly clear, here and elsewhere, that what I am referring to is that the mathematical structure of the fundamental theories we are using (the SM and GR) have not changed. Do you deny this?

      "I also disagree that what is needed is to throw out everything we have done so far,"

      I never said anything remotely like that. Stop fabricating things I did not say.

      "progress in collider physics will be slow and if you don't have the patience for it..."

      Look, I interviewed Nima for my book. He said pretty much the same thing there, though we did not talk specifically about the new collider thing. As I explained many times already, of course physics is hard and of course you need patience. This, however, does not explain why physicists continue to make wrong predictions, even though their methods are demonstrably not working.

      "I also think you are deluding yourself in thinking that progress will be fast and easy..."

      I never said anything remotely like that. Stop fabricating things I did not say.

      "I will repeat what I already said there, which is that this is just not true, breakthroughs can come in a variety of forms, and there has been a number of discoveries that did not come from resolving previously known inconsistencies, like the Rutherford experiment, the CMB discovery, etc."

      Right, we talked about that before and I explained to you already that this "objection" is nonsense. You simply refuse to listen to my response. Here we go once again: If an experiment finds a discrepancy between data and previously existing theories (or if such theories don't exist) and in response theories must be revised, then it's experiment-led.

      This does not imply, as you have falsely and incorrectly stated repeatedly that those inconsistencies must have been previously known. Of course it is not necessary that they were known before.

      I am merely pointing out what I think should be patently obvious: In case that you *do* know such an inconsistency, then you should closer study it. That is a promising way to progress.

      "I did not misunderstand you, but astrophysical searches have also come up empty so far..."

      You clearly do misunderstand me. Once again, I am *NOT* talking about searches for specific particles that we do not even know exist. I am saying we need better data on the already known discrepancies.

      "Please explain to me how you quantify that these discoveries are "more plausible"."

      They are more plausible because quantum gravity is necessary to resolve an internal inconsistency in the existing theories.

      "Furthermore, science funding does not work as a priority queue where only the "most promising" experiment gets funding"

      Money that is spent on a bigger collider will not be spent on something else.


      Delete
    2. (I did not publish your second comment since it seemed to be identical to the first. It sometimes takes me a while until I come around to approve comments in the queue, sorry.)

      Delete
    3. Fx,

      You said, "First of all I think it is not really true to say there has been no progress [in fundamental theories of physics such as elementary-particle physics in the last forty years]."

      This seems to me a crucial issue: I first got involved in high-energy/elementary-particle physics as an undergrad in the summer of 1973 working on the Barish-Sciulli experiment, at just about the time that QCD was first formulated. Weinberg-Salam and the Higgs mechanism had been worked out a few years earlier.

      So... what progress has occurred since then?

      Basically, we have "filled out" the Standard Model with all the quarks, and, just a few years ago, the Higgs particle, all of which I was learning about way back in the 1970s!

      I'm afraid that my own personal life experience really does feel as if we knew it all back in the 1970s and we have just filled in the blanks since.

      You also said, "I also disagree that what is needed is to throw out everything we have done so far, call it a day and do something else."

      But, very often that is the right thing to do: refusing to do that is known as the "sunk-cost fallacy."

      It does seem to me that part of the debate between you and Sabine hinges on whether cost matters. The Rutherford-Geiger-Marsden experiment was, after all, quite cheap. Even if it had been a long shot, it would have been worth it, and, in fact, it was likely to produce an interesting result of some sort, perhaps the first experimental confirmation of the "plum-pudding model," maybe something more interesting (as actually happened).

      But, suppose that costs are such that you can fund one huge experiment such as the next generation collider with a probability p of interesting results, or a hundred experiments that are much cheaper, like the Rutherford-Geiger-Marsden experiment, each with probability of interesting results of only 0.1 * p.

      Which funding choice is the wise one?

      Delete
    4. PhysicistDave,

      There has been a lot of progress in actually understanding the standard model and how to make predictions from it, for example. In the 1970's it would have been completely unthinkable to perform the analyses currently being done at the LHC, because most of them rely on calculations that we only know how to do since a couple of years.

      Perhaps some may think that these developments are not "foundamental" enough, it is true that not much has changed in what makes up the standard model (but not nothing, neutrinos have mass for example). But I think it would be very mistaken to discount this as purely technical advances. Among other things, in order to calculate a cross section up to two or three loops, and resum logarithmically enhanced terms in problematic kinematic regions, a really huge amount of progress had to be made in understanding amplitudes and how divergences appear, as well as how to compute and combine all these separately infinite integrals into a finite result.

      As for cost, it is true that a debate can and should be had on how much is too much and whether an FCC is still a reasonable expenditure for humanity. But this should be a technical discussion relying on quantitative arguments and facts, not on a few high-level comments on the promise of the field. I don't think your estimates of probability provides any helpful guidance for example: it is impossible to know what really is the coefficient that you set to 0.1, is it 10^-5 or 10; the success chances of some of these hundred experiments are most likely to be correlated; how do you factor in the fact that a collider can do actual measurements of new couplings that these other experiments can't do? By changing your priors you can pretty much defend any position you want.

      Delete
  7. Hi Sabine,

    I think you are misinterpreting what Fx said about the public. Clearly it is public money and people have to know where it goes. Moreover, we have to do our best to explain everyone what we are trying to do and why.

    On the other hand, it would be very naive to say that the "public" who does not know anything about fundamental physics should be able to understand what is worth investing their money on. It is like if pharma companies were asking the public if one should invest in this particular molecule or another... it simply does not work like that and cannot work like that.

    Let's try to avoid to fall into this way of thinking that "everyone has a saying on anything". This is like anti-vax and flat-earth movements have come about. I hope you don't like that either, Sabine...

    Also, on the science part, you cannot say that Dark Matter searches have a better chance of revealing something. You are contradicting yourself... you said there might not even be any dark matter at all, and the continuous null results of DM searches experiments are definitely more striking than the ones of collider physics...


    Lorenzo

    ReplyDelete
    Replies
    1. Lorenzo,

      No one expects that non-experts understand the details of a research proposal. But scientists should be able to explain what the benefit is of doing their research to begin with.

      At current particle physicists are not even able to explain the benefit of their research to experts.

      "Let's try to avoid to fall into this way of thinking that "everyone has a saying on anything". "

      I did not say anything like that. I am merely pointing out that if you want $20 billion dollars from taxpayers, you better have a good reason why you ask for that. Particle physicists do not have a good reason.

      "you cannot say that Dark Matter searches have a better chance of revealing something. You are contradicting yourself... you said there might not even be any dark matter at all, and the continuous null results of DM searches experiments are definitely more striking than the ones of collider physics...

      You are misunderstanding what I say. I most definitively do NOT say that we need further direct searches for dark matter particles which we do not even know exist. What we do need is more details on the observations that are commonly attributed to dark matter. Because, indeed, it may not even be a particle.

      Next time, before you accuse me of "contradicting myself" maybe consider that you simply did not understand what I said.

      Delete
  8. Hi Sabine,

    there is (at least) one point I do not understand well.

    Sabine: …“you want to make progress in our understanding of the foundations of physics” …

    => What do you mean with this?

    Brian: … “indication of where there will be new physics at high energies.” …

    => Brian speaks about high energies, not you. You speak about foundations. Then, surprisingly:

    Sabine: “I totally agree with that. The more precisely you measure, the more sensitive you are to the high energy contributions. But still there is no good reason right now to think that there is anything to find, is what I’m saying.”

    => So you agree with this statement about “high energy contribution” and I (and I guess everyone else reading) understand(s) new particles and fields. (Standard wording).

    Here, I think you have just been conned and driven to high energy, which – as far as I know – may have nothing to do with new or better understanding of the foundations of physics. (I understand and I mean physics, that is to say the real world).

    Note that I agree with your conclusion. But if you want taxpayed organizations to work for any change (in particular to agree that something cheaper is worth money), I think you do not have a chance. (Though I do not believe in quantum gravity; I think it is the opposite).

    Best,
    J.

    ReplyDelete
    Replies
    1. akidbelle,

      I am afraid I don't understand your comment.

      What I mean with progress in the foundations of physics is that we find theories that have higher explanatory power than the presently used ones (SM and LCDM). Or, if you believe there's a theory of everything, maybe it would only be one theory.

      Going to high energies implicitly means going to shorter distances. This is a good way to explore unknown parameter space and that's what colliders do. This is all well and fine. What I am saying is that the way it currently looks, there may be more than 12 orders of magnitude in energy before there is something new to see again. Hence, given the current information building a larger collider is not the most prosiming thing to do.

      I don't understand your going on about particles and fields. The reference to "high energy contributions" is actually more general than that. It encompasses pretty much everything that appears in an effective field theory framework, and that could be handed down from a theory that uses entirely different mathematical structures than particles and fields. String theory may be an example.

      Delete
  9. Is spending 20B on a new collider a good investment?

    Your position is no. Showing logical flaws in arguments in favor of the investment does not prove your case.

    Don’t you need to show a specific alternative set of investments that provide a better return (at no greater cost) to prove your case?

    ReplyDelete
    Replies
    1. Stuart,

      "Your position is no. Showing logical flaws in arguments in favor of the investment does not prove your case."

      If you want $20 billion you need a good reason. Demonstrating that the arguments from particle physicists are logically flawed demonstrates the absence of such a good reason and hence proves my case.

      Besides, I have said this many times already, and I also say this in the podcast, that we do have alternatives: The money would be better invested in studying those cases where we have reasons to think something new is to be discovered. This is for example astrophysical observations of dark matter, and in quantum gravity or quantum foundations. Such experiments are both less costly and have better motivations, hence more benefit for less cost. They are better investments and should have higher funding priority.


      As I have said many times before, I do not expect everyone to agree with my argument, but at least I do have an argument.

      Delete
    2. You absolutely have a good argument, one I tend to agree with.

      ‘Demonstrating that the arguments from particle physicists are logically flawed demonstrates the absence of such a good reason and hence proves my case. ’

      All it demonstrates is that their arguments are flawed, not that the underlying claim is wrong (a poor debater’s claim can be right even if their argument is wrong). If you invoke logic, yours must be impeccable.

      Delete
    3. Stuart,

      I am not sure why we are disagreeing, so let me be clear. The claim (made by particle physicists) that "We have good arguments for building a next larger collider." can be proved wrong by demonstrating that those arguments are not good.

      This is what I have been saying.

      You may instead have been saying one of two things. First, demonstrating that one of their argument is wrong does not mean they do not have any good arguments. That is correct. I have in this post not addressed all arguments, but have done so in earlier blogposts. If you think I have forgotten to address an argument, please let me know.

      Second, you were talking about a different claim which is "Building an next larger particle collider is a good investment."

      In that case it is correct of course that demonstrating that particle physicists' argument for the collider are flawed does not demonstrate that it is not a good investment. Note however that I have furthermore also provided arguments for why it is not a good investment. If you want me to go through this once again, please let me know.

      Delete
    4. Stuart's argument seems to imply that the default position should be that they get their 20B unless someone else can come up with a better use for that sum of money. But if you're asking for 20B, the onus is on you to prove that you need it more than anyone else and that you'll be making the best use of that fortune!

      To me it is clear that the law of diminishing returns has become effective in experimental high energy physics research. Greater and greater sums of money are needed for smaller and smaller potential returns. The society has to stop and think if the priorities need to be shifted elsewhere.

      Delete
  10. please explain why the case that we would discover something in quantum gravity is strong

    ReplyDelete
    Replies
    1. A theory of quantum gravity is necessary to resolve internal inconsistencies in the existing theories. It is a good prediction in the same way that the Higgs was a good prediction. The same is not the case for example for supersymmetry. Supersymmetry is nice but not necessary. Similar considerations apply to extra dimensions or grand unification: Nice, but not necessary. Quantum gravity is not of this type.

      Delete
    2. Sorry - perhaps I should clarify. I understand in general why a theory of quantum gravity is needed and am not suggesting that people don't work on it. What I am interested in is why , at this time, there is any belief that investing money in it will yield any experimental results to help us understand what this theory is.

      Delete
    3. kbot,

      I explained this here, see esp the paragraph that begins with "Besides strong space-time curvature..." which contains further references. Please let me know in case that doesn't clarify it and I'll be happy to explain.

      Delete
  11. Sabine,

    Do you have the full recording without editing?

    The fade at the end cuts you off when you try to explain examples of some promising research that is starved or in need of funding.

    It is ironic that a 2010 Guardian article quotes Brian Foster "seriously contemplating" leaving England to go to Hamburg to obtain 4.3 million pounds in funding over 5 years. "There is nothing remotely as generous in the UK."

    https://www.theguardian.com/education/2010/sep/30/scientists-threaten-leave-britain-funding

    Even before this, Foster is mentioned in an article discussing budget cuts and the political nature of deciding which experiments "take priority":

    https://physicsworld.com/a/no-extra-cash-for-uk-physics/

    Quoting from the author of the article:

    "the STFC ( Science and Technology Facilities Council ) has completed a major consultation with the scientific community to help determine which projects should take priority in the face of inevitable cuts."

    Can we just call Foster a persistent lobbyist?

    And that part of that lobbying is denial that the lobbying exists. That fighting/competing over limited resources (researchers and money) is an integral part of his job.

    The other part of job is convincing people that "other researchers" have plenty of money and creating the narrative that skeptical voices are simply wrong.

    ReplyDelete
    Replies
    1. RK,

      I only have a recording of my part of the conversation. Yes, the discussion is cut in various places. I believe I was referring there to SKA not having received funding for the full proposal, which is a direct counterexample to the claim that we are supposedly pushing all frontiers and everyone will get funding etc, as particle physicists want you to think. It's simply not true.

      Delete
  12. (A rant): "dark matter must be made of particles of some sort" = a century they argued that ether must have some mechanical properties. Yes, I know search for dm is totally not the same because. High precision measurements of the Hubble constant, depending on method, seem to produce 2 different values, but that's another fish. etc etc

    ReplyDelete
  13. In industry, before there is a big investment made in a project, a pilot project is setup to test the basic assumptions that underpin the requirements of that expensive project.

    For the next big particle collider, this type of pilot project could be done and in fact is being done right now by the Alpha Magnetic Spectrometer, or AMS, that was launched into space in 2011 and is now housed on board the International Space Station.

    This project has been using naturally produced particles with some having maximal energies far higher than anything that can ever be produced on the surface of the earth and there have been no indications of any physics: no supersymmetric particle, no extra dimensions, no dark matter particles, or more massive Higgs particles, nothing that would justify the construction of a big new particle collider that has been put forth by the advocates of this new super expensive collider.

    In order for these advocates to come up with a proof of principle to justify this massive project, they might be well served to imagine a way, a less expensive way, to show that this project is worth the cost of its construction and not leave the build decision as an article of faith.

    ReplyDelete
    Replies
    1. Axil,

      The high energy tail of cosmic ray collisions currently reaches comparable center of mass energies to those produced in colliders. However, the reconstruction of cosmic ray events is more difficult because you don't know for certain what was colliding in the first place. Also, to state the obvious, the higher the energy the smaller the flux.

      Delete
  14. I read Nima's interview. I think he has some interesting points on this. In particular I tend to agree with his discussion on the Higgs field as something odd, a scalar with only mass and apparently nothing else except this odd property of forming a condensate that binds three of its Goldstone bosons to W and Z and fermions.

    https://cerncourier.com/in-it-for-the-long-haul/

    ReplyDelete
  15. While I enjoy reading this discussion, I am a bit surprised to find myself relegated to the public that needs not to worry our pretty little heads about such things like ultra big colliders. I remember how the public was solidly behind the space program to the moon. Just as they were for the Hubble space telescope. There are more, but I just antw to say one more thing. We live in an era where vast swaths of people do not believe in global warming, vaccinations are being skipped, and conspiracy theories are out of control. Writing the public off is something scientists have done for a very long time. However, now they are reaping distrust. If you want to spend $20billion on a bigger collider then you better have $20billion questions you want answered. Even better would be a look back over 40 years of high energy physics and show a running total of information gained versus what it has cost. Then do a cost benefit analysis.

    ReplyDelete
  16. Hello. I am a lay person. Recently I have been questioning and pondering the actual ("practical") predictive power of our existing theory (i.e. Standard Model). I have a growing suspicion that most non-physicists, even those interested in physics, have a greatly exaggerated sense of the power of current theory in practice. After all, we looking for a Grand Unified Theory or a Theory of Everything; when we find it, "we should be able to predict everything", right?

    For an example, a follower of mine on Quora wrote "There is an ominous implication here that a Theory of Everything presumes to explain all phenomena ... WHO would even bother to seek a career in the discipline of physics when ALL the questions have already been asked, and all the answers have been provided."

    I tried to explain that in practice, the Standard Model can't even post-predict that deuterium is stable and tritium is not; even a system with just 2 or 3 nucleons lies beyond what is practical. We don't have to worry about "predicting all phenomena" quite yet. ;-)

    But in turn, my argument is a curse. It seems to me that if there was a better understanding of how little predictive power research at the cutting edge of HEP was actually "buying", it would be much more difficult to argue for funding. Nobody would pay $20 billion dollars for a theory that can't predict from first principles the behavior of 3 nucleons.

    Of course, the hope is that additional research will lead to a theory that is able to make better predictions; but in practice, is there any reason to believe this?

    Or perhaps I'm just way off base. Sabine, do you have any thoughts about this?

    ReplyDelete
    Replies
    1. Jeff,

      Well, what you say is correct & I also explain this in my book. While the standard model is the theory for all interactions besides gravity, in practice physicists cannot presently even derive the properties of atomic nuclei from it.

      The reason is that even though the theory is known, the equations cannot be solved. I can go into this in more detail, but I don't think it matters.

      For this reason, even if we had a "theory of everything" that would combine all the known interactions (including gravity), we still would not be able to actually calculate everything.

      This is a point which has been made many times, it is entirely correct, and yes "theory of everything" is a misnomer.

      "Nobody would pay $20 billion dollars for a theory that can't predict from first principles the behavior of 3 nucleons."

      As a matter of fact taxpayers have spent much more money on such theories already, so your statement is simply false.

      Delete
    2. What Jeff's question pertains to where the nuclear binding force comes from with respect to QCD. This involves some Lie algebra and the Cartan decomposition of Lie groups and the algebras that generate them. The Lie group for QCD is SU(3) and this can decompose into SO(4) ~ SU(2)×SU(2). At lower energy we tend to have a symmetry decompose or break. So we then have embedded in SU(3) two SU(2)s, one for chirality and the other for isospin. This product of two SU(2) is then broken by the effective Goldstone bosons of the the isospin so there is a quotient group G/H = K. for H the isospin. This means the resulting quotient is "mod isospin" or that it is a symmetry of isospin and so we have SU(2)×SU(2)/SU(2)_{iso}. Interestingly leaves us with an SU(2) that conserves isospin. This is the old theory of nuclear force or gauge field for nucleons.

      As a side comment we can rotate the isospin and chirality to mix them, similar to a Weinberg angle. This results in a gauge field that has some chirality, and is connected to the Peccei-Quinn theory with the axion. So far this has not born fruit in experiments.

      The problem is that though we can understand how the nuclear binding gauge field occurs from QCD this does not mean it is easy to work. For instance, with the deuteron this is a two body problem and fairly amenable to calculation. The tritium is a three body problem and not so easy. Computing the structure of nuclei is very difficult. This SU(2) gauge field is nonabelian, which makes it tough, and then throwing many body complexities onto that is even more difficult.

      Physics has these domains of applicability. While atomic physics is better understood than the physics of the atomic nucleus, we have not quantum mechanical prediction for the occurrence of large molecules. The protein folding problem illustrates the limits of our abilities here.

      Delete
    3. Anyone who thinks that even classical physics has explained/predicted all the phenomena within its domain should look up "Striations in Kundt's tube".

      Delete
    4. Thank you. I did read your book, but I guess some parts didn't completely sink in on me ... perhaps because I didn't want to believe it was that bad.

      Delete
    5. In fact, the argument ought to go the other way. The particle physicists have fallen into the trap over the years of emphasizing what Sabine calls "foundational" issues while exaggerating the predictive capabilities of the SM, even though the QCD aspects are really not that well understood and the LHC did find new things in that department. So we are now in the somewhat absurd situation where resolving basic questions about the proton isn't "fundamental." A more modest and objective presentation of the state of knowledge would have made it easier to convince the broader scientific audience and the public that the "mundane" measurements that the new collider will certainly be able to make are worth the money.

      Delete
    6. SRP,

      Indeed, an alternative history might have looked like this. This was the reason for my earlier comment that particle physicists dug their own grave by hyping the LHC as a machine to answer "big questions".

      Most depressingly, though, they still haven't learned the lesson. They are still trying to push the "big questions" angle.

      As I have said before, the only thing that will help particle physicists at this point is to:

      (a) make clear, public statements acknowledging that their predictions for BSM physics were all wrong instead of pretending it didn't happen
      (b) take steps to prevent further hype and to counteract communal reinforcement that results in such bad methods becoming accepted practice

      Unfortunately, it looks like this will not happen simply because they do not want to believe that their field even has a problem.

      Delete
    7. Sabine,

      I agree that there have been some overrated announcements on what LHC can do - like, Dimopoulos (or was it Arkani-Hamed ?) saying "Either we'll find supersymmetric particles at LHC or it's the proof of the existence of the Multiverse."

      But seriously, LHC is producing a string of very convincing and important results, for example the couplings of different particles to the Higgs, which show the validity of Standard Model calculations over many orders of magnitude. Digging a grave is something else !

      It's always nice when one can announce a discovery, but the scientist's work is often mundane: increasing precision, improving exclusion limits, looking for discrepancies.

      LEP did not make a single discovery, but it was the collider which firmly established electroweak theory and QCD as "The Standard Model" and it even made a prediction about a light Higgs, which it failed to discover by a few 10s of GeV.

      LIGO made one discovery (gravitational waves), and now it's used to disentangle the dynamics of black hole and neutron star mergers, very useful indeed, but now it becomes routine progress in a new window to the universe.

      If an FCC hadron-hadron collider would not make any discovery (for which no-one can give the probability, as we don't know the probability distribution function masses of particles that we haven't discovered yet), it would still push the limits of our understanding of particle physics, by less spectacular, but nevertheless important results for the advancement of understanding the universe.

      Delete
    8. Thomas Otto,

      There have been many such "overrated announcements". I have collected a selection here.

      Unless particle physicist make a public statements admitting that those proclamations were wrong and explain what steps they take to prevent hype and group-think in the future, I see little chance they'll see a next collider.

      Delete
  17. "The real tragedy is that there is absolutely no learning curve in this exchange."
    Isn't this true for both sides in this exchange?

    ReplyDelete
    Replies
    1. quantphys,

      To my best knowledge, I have considered and addressed every argument that was posed to me (in dozens of repetitions). If you think I have missed one, please let me know.

      I do not get the impression that any particle physicist cares to even think about my explanation why their arguments are (obviously) flawed, and they simply continue to make them, hence my comment.

      It is somewhat perplexing in that it should be in their interest to make good arguments.

      Delete
    2. "I do not get the impression that any particle physicist cares to even think about my explanation why their arguments are (obviously) flawed, and they simply continue to make them, hence my comment."

      Car driver listening to the radio while driving... "People, be careful, we got information of one car driving on the wrong side of the road!"... the car driver thinks..."hey!... it's not only one car on the wrong side, I see all of them doing it!"...

      ... hello Sabine, how's that car trip of yours??? :-)

      Delete
  18. Jeff, I agree very much with you comments. Even at the level of non-relativistic quantum mechanics, 'we' can't predict that much. Molecular structure calculations require a great deal of approximation - otherwise most experimental chemistry could be performed on the computer.

    I think physicists need to think about the real use (or lack of use) of the data they propose to collect. Even to predicting the world around us.

    There are vast numbers of species of insects, bacteria, viruses, etc yet to be discovered. Can those who propose spending $20 billion (say) on the FCC, explain why it might not be better to spend it on seeking out the creatures yet to be discovered?

    ReplyDelete
    Replies
    1. "Can those who propose spending $20 billion (say) on the FCC, explain why it might not be better to spend it on seeking out the creatures yet to be discovered?"

      That's a pretty silly/nonsense question.

      Delete
    2. Roberto,

      I see nothing silly in suggesting that there may be areas of science than would produce far more results with $20 Billion.

      The spinoff from the discovery of new bacteria or fungi could well include the discovery of new antibiotics, which are badly needed right now because of the rise of resistance to existing antibiotics.

      Delete
    3. Indeed, the question whether their research is worth the money to begin with is a question that every particle physicist should think about and respond to politely with (hopefully) good arguments rather than simply declaring it as "silly".

      Delete
    4. @David Bailey

      "I see nothing silly in suggesting that there may be areas of science than would produce far more results with $20 Billion."

      Only if you define "far more results". Results is what, for you? If it is results in particle physics I doubt that spending 20 billion on cataloging more butterfly species would help. See why it is silly?

      "The spinoff from the discovery of new bacteria or fungi could well include the discovery of new antibiotics, which are badly needed right now because of the rise of resistance to existing antibiotics."

      Sure. But probably it wouldn't. Big pharma is spending already way more than the 1 billion/year that the FCC would demand:

      https://www.statista.com/statistics/309466/global-r-and-d-expenditure-for-pharmaceuticals/

      >200 billion dollars/year...

      See why your statement is silly?

      Cheers.

      Delete
    5. Roberto wrote,

      ". See why it is silly?"

      I suppose the obvious answer is, only if you define the word, 'silly'!

      Since a lot of new antibiotics have come from fungi, I think my proposal would have a reasonable chance of producing something really valuable - look at the table entitled "Some clinically important antibiotics" here:

      http://archive.bio.ed.ac.uk/jdeacon/microbes/penicill.htm

      I think we all know that your proposal would very likely yield less than even the LHC. and mine was a zero-effort approximation.

      I feel the (really extraordinary) truth is that rather than HEP yielding some extraordinary foundation to the rest of science, it has tailed off over decades into complete irrelevance! Nothing that has been discovered about the Higgs is remotely likely to be incorporated in any practical calculation at all. Contrast that with Faraday/Maxwell's results, that still dominate many of today's industries.

      It was people like Faraday and Maxwell that earned physics its prominence, not Carlo Rubbia!

      My gut feeling is that somewhere physics took a wrong turn (the topic of one of Sabine's earlier blogs), but whatever has happened, modern HEP isn't worth 20Bn.

      Delete
    6. "My gut feeling is that somewhere physics took a wrong turn (the topic of one of Sabine's earlier blogs), but whatever has happened, modern HEP isn't worth 20Bn."

      Your ideologically-founded prejudicial conclusion. Nothing more.

      Saying that since Maxwell nothing much happened is sillier than your earlier statement... it borders madness!... what planet do you live on?

      Basically all electronic devices of today work thanks to post-Maxwell physics, and most of them rely on particle accelerators for their development and fabrication, just to name one obvious and often understated fall back/spin off of research on the field.

      Without prior HEP research, most of it done at Cern, there would be no neutrino physics (I mean accelerator-based one), even no gravitational detectors, so what the heck are you talking about?
      Money spent at Cern and similar labs all over the world is repaid many times over. That's a fact.

      20 billions over 20 years is peanuts for the 30-ish member/associate/etc states composing Cern, that's also a fact... and if fungi are your interest and need further funding then there's appropriate funding agencies dealing with health care, not physics!... which should be invited to act.
      Thinking that taking money off particle physics will make other fields of research better funded is the mother of all silly statements I've read so far... and the proof is that your statement is shared by Sabine, who understand nothing about big science projects and their specificities.

      She's capable of only using straw man arguments, and smearing of a whole community.

      Cheers.

      Delete
    7. Roberto,

      You write

      "She's capable of only using straw man arguments, and smearing of a whole community. "

      Please provide evidence for a straw man argument that I have made.

      "Thinking that taking money off particle physics will make other fields of research better funded is the mother of all silly statements I've read so far... and the proof is that your statement is shared by Sabine, who understand nothing about big science projects and their specificities."

      I have not made such a statement or supported such a statement. Stop fabricating things I never said.

      Delete
    8. @Sabine

      "Please provide evidence for a straw man argument that I have made."

      I wouldn't know where to start from, Sabine.
      Like trying to put the out-of-contest opinion of one major particle physicist against another one?

      Your modus operandi is clear.
      Smear&lie: any credibility you had is lost.
      But, folks!... don't believe me!.. Go buy her book, and you'll see the light.

      Yeah, sure.

      Delete
    9. Roberto,

      Your accusation that I have not provided context for the quotes, is, as everyone can see, false.

      Since the quotes are not an argument, they are not a straw man argument.

      Once again you merely demonstrate that you are unable to argue to begin with.

      Delete
  19. Hi Sabine,

    Ok I will only come back on two things, otherwise we will go in circles again.

    On the topic of quantum gravity: you can't just wave your hands and say there is an internal discrepancy between theories and so a discovery is more likely. No sh*t there is are things we don't understand at the planck scale. The problem is that it is hard to probe and there is no guarantee we will be able to do it successfully. Sure there are some promising experiments currently operating or being designed, I am not enough of an expert on the subject to be able to really way in with an informed opinion on how good the odds are, but at the moment I can't see any reason to be anything more than cautiously optimistic. What concretely would you do with billions of dollars anyway? It's not like there are brilliant ideas out there waiting to be picked up, if someone has a good idea, it is pretty easy to get funding for it, and throwing money at the subject is not going to give us experimental measurements of quantum gravity within X years. By contrast, collider experiments are a pretty close to a sure thing insofar as if there is anything within their reach, the odds are good to find it.

    "Right, we talked about that before and I explained to you already that this "objection" is nonsense. You simply refuse to listen to my response. Here we go once again: If an experiment finds a discrepancy between data and previously existing theories (or if such theories don't exist) and in response theories must be revised, then it's experiment-led.

    This does not imply, as you have falsely and incorrectly stated repeatedly that those inconsistencies must have been previously known. Of course it is not necessary that they were known before. "


    Well yes, and the FCC would be a prime candidate for experiment-led breakthrough. If this type of discovery counts for you as resolving an inconsistency between data and theory, fine, but then your argument doesn't make any sense because it is also in favor of building a new collider. If you are saying what I undestood to be that historically discoveries tend to come from looking at places where we already know there is a discrepancy, then I am just saying, this is simply not true, there are plenty of cases of experiment-led breakthroughs where we had a priori no reason to expect to discover something.

    ReplyDelete
    Replies
    1. Fx,

      "you can't just wave your hands and say there is an internal discrepancy between theories and so a discovery is more likely"

      I do not just "wave my hands", those are the theoretical predictions that what worked well historically. If you wanted to, you could quantify this and derive a prediction from it, but I think this would be an overkill.

      "What concretely would you do with billions of dollars anyway?"

      I understand you don't really want an answer to this I, but I will give you one anyway. The first I would do is finance a study group to evaluate the prospect of experiments in the foundations of physics. You probably know, but some readers may not know, that the CERN study for plans of a future collider is supported by 11 million Euro.

      If you complain that other people (eg: me) do not have competing proposals that are worked out at a similar level, then you are primarily drawing attention to the fact that in science the rich get richer. Give me 11 million Euro and 4 years and I'll be back with a stack of proposals detailing what I recommend we do with the billions of dollars.

      "then your argument doesn't make any sense because it is also in favor of building a new collider..."

      That I distinguish between theory-led and experiment-led breakthroughs is not an argument for or against anything, it is just a terminology that I find useful.

      "If you are saying what I undestood to be that historically discoveries tend to come from looking at places where we already know there is a discrepancy, then I am just saying, this is simply not true,"

      This is *NOT" what I am saying. Once again, I am saying that the more expensive experiments become, the more careful we have to be when deciding which experiment to make. This is not the 1920s. We have tried experiment-led breakthroughs in the past 40 years - with zero success. Therefore, I say, do the obvious: Collect more information on the already known discrepancies between theory and experiment, and focus on those theory-led predictions that have historically worked, that are those based on resolving internal inconsistencies.

      "there are plenty of cases of experiment-led breakthroughs where we had a priori no reason to expect to discover something."

      I have already agreed on this multiple times. I have also told you several times already that this is not an argument for building a larger collider in particular. "Just look" is an argument that can be made for any experiment that probes new parameter space. It is not helpful to make decisions about where to direct efforts.

      Delete
    2. "I do not just "wave my hands", those are the theoretical predictions that what worked well historically."

      They only work in the sense that we know something is there to be found, but they provide zero guidance in how difficult or likely a discovery of these effects is. For quantum gravity there are also good reasons to expect that this will be in fact extremely difficult.

      "I understand you don't really want an answer to this I, but I will give you one anyway. The first I would do is finance a study group to evaluate the prospect of experiments in the foundations of physics. You probably know, but some readers may not know, that the CERN study for plans of a future collider is supported by 11 million Euro."

      Thanks for replying, in fact I did not know this about the FCC study group although that seems reasonable. I would also be totally in favor of similar studies for QG and DM experiments, in fact I find it hard to believe that such a thing would not exist already. I know for example of several programs oriented around quantum sensors both in the US and in the UK with funding in the tens of millions each.

      "That I distinguish between theory-led and experiment-led breakthroughs is not an argument for or against anything, it is just a terminology that I find useful."

      I have no problem with that. But I will quote below your first use of this phrase, which seems to me quite a bit different than what you are now saying:
      "Historically, breakthroughs have come from resolving inconsistencies between theory and experiment or from resolving internal theoretical inconsistencies. Therefore, the areas where such problems exist are the ones we should focus on to make progress."

      What I said is that this is not true: discoveries have not always come from resolving known inconsistencies. If you then tell me you meant to include experiment-led breakthroughs with no prior known inconsistencies in the first part of your sentence, then this becomes an argument neutral or supportive of the FCC. Now you are saying that this is not the 1920s and we have to be more careful in how we invest our money, but this is a completely different argument! Who is guilty of Motte and Bailey now?

      As for "just look" not being a good argument: if this was the only argument in the case for the FCC, I would say maybe you are right and it is not good enough. But it is not the only argument, measuring the Higgs sector will yield results that will add several pages to any future textbook on fundamental physics, whatever the outcome.

      Delete
    3. Fx,

      I believe I finally understand where our misunderstanding about theory-led vs experiment-led is coming from. You write

      "I have no problem with that. But I will quote below your first use of this phrase, which seems to me quite a bit different than what you are now saying:
      "Historically, breakthroughs have come from resolving inconsistencies between theory and experiment or from resolving internal theoretical inconsistencies. Therefore, the areas where such problems exist are the ones we should focus on to make progress."

      What I said is that this is not true: discoveries have not always come from resolving known inconsistencies. If you then tell me you meant to include experiment-led breakthroughs with no prior known inconsistencies in the first part of your sentence, then this becomes an argument neutral or supportive of the FCC. "


      I am still saying the same thing. The point you are missing is that making a measurement which conflicts with previous theories in and by itself is not a breakthrough. The breakthrough only happens once you resolve that discrepancy by revising the theory.

      I already said that this is not an argument for or against the FCC. It is merely terminology. Hence, it is correct -and I have already told you several times it is correct - that this is neutral about the FCC.

      What I am saying is that if experiments become very costly we have to be careful where to invest the money. "Just look" has not worked for BSM physics for 40 years. We should therefore take a clue from history and focus on resolving the known discrepancies between theory and experiment ("experiment-led", eg dark matter) or internal inconsistencies in the theories.

      "As for "just look" not being a good argument: if this was the only argument in the case for the FCC, I would say maybe you are right and it is not good enough. But it is not the only argument, measuring the Higgs sector will yield results that will add several pages to any future textbook on fundamental physics, whatever the outcome."

      That's all fine with me. As I have said many times, measuring the Higgs self-coupling is all well and fine. You only have to convince taxpayers that that's worth $20 billion.

      Delete
  20. Sabine,

    You have been very actively discussing the short comings of the proposed "Future Circular Collider" (FCC) as of late.

    Do you know (and are allowed to tell us) why the big push for FCC started recently? I ask because my impression was that the next big collider was to be a "International Linear Collider" located in Japan.

    Can you speculate on of the two projects which has more scientific merit? I'm asking cause I don't know this is not intended to be a plug or a trap I really am this ignorant of the physics/politics here. I suspect the answer to my question is a CERN is trying to avoid lay-offs thing but I really am cautious of simple explanations like that.

    Also thank you for your continuing efforts in preventing the waist of public and private funds on projects of dubious efficacy.

    J*

    ReplyDelete
    Replies
    1. Jonathan,

      Planning and building a large collider takes a lot of time. If particle physicists want to have a next larger collider in operation by the time the scheduled LHC runs (with the HL upgrade) are over (late 2030s), they need to get started soon. It is thus not so surprising that the proposals (from CERN, China, and Japan) are presently all at similar stages (design plans, gathering community support).

      The current "big push" for the FCC that you see right now is for all I know merely due to their study being completed and published in time to for the European particle physics strategy meeting, some time this Spring.

      I don't know anything about the ILC that you wouldn't also know, which is, roughly speaking, not looking all that great.

      The collider projects all have scientific merit. This really isn't the point of debate. The point of debate is whether the science justifies the investment.

      Delete
  21. It's interesting to see how much these folks are struggling to explain themselves -- certainly shows that they are not used to being challenged. I actually think they have a pretty good case. If you look at the past 30-40 yrs (the part of history I personally experienced) then fundamental physics (broadly defined) has seen a series of discoveries. W/Z bosons, the top quark, flatness and accelerated expansion, neutrino masses, the Higgs, gravitational waves. Many of these were "expected", but some were not. These discoveries involved a broad portfolio of experiments, colliders, underground detectors, precision frontier, satellites, etc. This is a history of stunning, unbelievable success. Indeed, I'm occasionally a bit of a cynic myself, and these are more discoveries than I truly expected to see in my life time. If I look at this history I conclude that we should continue with a broad portfolio of experiments, and if at all possible, that should include colliders. I don't see how you argue at this point for new colliders at green field sites (say a linear collider in Japan), but to try and find a follow-up project at an existing lab like CERN makes a lot of sense to me.

    ReplyDelete
    Replies
    1. Thomas,

      If you look at the past 30-40 years, then the standard model has not been complete. It is now. Time to wake up.

      "but to try and find a follow-up project at an existing lab like CERN makes a lot of sense to me..."

      I find it interesting you would make such a statement without delivering a glimpse of an argument for how the investment is justified after you point out that particle physicists "are struggling to explain themselves."

      Whether or not such a follow-up project makes sense depends of course on whether the potential benefit justifies the investment. Please explain why you think building a next larger collider would be a good investment into progress in the foundations of physics.

      Delete
    2. Can't you still add a massive right-handed neutrino to the Standard Model, and still have it be a smallish tweak (the way that neutrino masses were)? I suspect the reason that nobody discusses this much is that the right-handed neutrino is probably much, much too massive to be discovered by any accelerators that we have a chance of building.

      (Another case of streetlight bias.)

      Delete
    3. Hi Peter,

      Yes, one can estimate the upper bound of such a right-handed neutrino and it's 2 orders of magnitude below the Planck scale or such (details depend on exact assumptions). In any case, it's a billion times higher than the energy that the collider would be able to test. The particle could be lighter but no reason why it would be. That's the situation.

      The terminology here is somewhat confusing, I think, because one might argue that the standard model is presently incomplete and one needs the right-handed neutrinos to complete it (or the neutrinos must be Majorana). But the way that the terminology seems to have evolved is that this would be physics beyond the standard model. This doesn't make a lot of sense to me because, well, consider we find those right-handed neutrinos then what do we call the model? The Beyond the Standard Model?

      But, well, terminology doesn't always make sense. So the present narrative is that the SM is complete but because of the neutrino masses we know there must be BSM physics rather than the SM is incomplete. Doesn't really matter, of course, which way one phrases it, one just shouldn't mix both terminologies.

      Delete
    4. This is all the seesaw reasoning with neutrino chirality. There is to my mind this persistent problem that a massive particle has a rest frame. For neutrinos this would occur for γ ~ 10^{26} or more. From a practical perspective a real observer can't boost themselves to such a frame. Yet in principle I see no objection. So I boost to this frame and left handed neutrinos with spin anti-parallel to its momentum can then flip to spin parallel to momentum. Does this mean I have generated a right hand neutrino with a huge mass?

      Realistically this means interactions with right hand neutrinos will be very high energy. I keep coming back to this elementary relativity QM question. How can an observer be Lorentz boosted so as to provide a light mass particle with lots of mass? To my mind there is something a bit odd with this.

      Delete
  22. The LHC was special, because it ended up being a no-loose proposition (but nobody could know that when it was planned, because the Higgs could have been discovered at the Tevatron or LEP). But most discovery experiments on my list were not no-loose propositions. A new collider is interesting, because it will explore physics that we have not looked at before (the Higgs sector etc). Is it worth the money? I don't know. But to me, the algorithm for deciding that is not so hard. If it threatens to consume the rest of the portfolio, the precision experiments, the satellites, etc, then we can't afford it. If it can be done keeping the breadth of the field intact, then absolutely we should do it.

    ReplyDelete
    Replies
    1. Thomas,

      What point are you trying to make by bringing up Tevatron, LEP, and the LHC? That maybe we didn't need all three of them? Be careful when making arguments from the past. Just because an experiments did (or did not) get funding doesn't mean it was a good (or bad) investment. That's just a different question.

      The relevant point is that, as you say correctly, particle physicists knew until 2012 that the standard model is incomplete. This is not the case any longer.

      "A new collider is interesting, because it will explore physics that we have not looked at before "

      The same can be said about any new experiment in the foundations of physics. They will measure something to better precision or at farther distances or for larger systems or for smaller systems, and so on.

      "But to me, the algorithm for deciding that is not so hard. If it threatens to consume the rest of the portfolio, the precision experiments, the satellites, etc, then we can't afford it. If it can be done keeping the breadth of the field intact, then absolutely we should do it."

      That's a terrible argument because it can be made regardless of how costly the experiment and regardless of how small its benefit. Just consider the collider costs $20 trillion and will do nothing besides measuring the mass of the electron. According to your argument it would still be "worth the money" and you would then just complain that taxpayers won't give you $20 trillion.


      Delete
  23. $20 trillion is a strawman. Again, the real algorithm is not so complicated (but difficult to implement). The total funding for fundamental physics has been fairly stable in recent decades, so the future funding envelope is (roughly) known. Given the total cost, construction and operation time scales, external contributions, etc, does an LHC follow-up wipe out the rest of the program? The problem is obviously that the overall scale of these projects has become so large that factor-2 mistakes can be devastating. But that is where detailed planing exercises come in.

    ReplyDelete
    Replies
    1. Thomas,

      It's not a strawman, I have merely demonstrated the absurd consequences of your argument. You now write:

      "real algorithm is not so complicated (but difficult to implement). The total funding for fundamental physics has been fairly stable in recent decades, so the future funding envelope is (roughly) known. Given the total cost, construction and operation time scales, external contributions, etc, does an LHC follow-up wipe out the rest of the program? "

      First, let me mention that this disagrees with the usual "no zero sum" argument. That's fine with me, just pointing out.

      Second, in the foundations of physics we always have more ideas than can get funded, so we have to make decisions. Resources are limited. To make these decisions you are proposing now that the conditions to fund a collider should be different to the conditions to fund any other experiment. Instead of having to explain why such an experiment is a good investment you say that particle physicists merely have to demonstrate that the total cost is less than the entire budget.

      I am not sure who you think will be convinced by this argument.

      Delete
  24. I would like to see a neutrino detector designed and built to detect where neutrinos are coming from by determining their direction of arrival, their energy, the length of their existence at the time of detection, their type (3 or 4 flavors), and their relative intensity. I would like to know from this neutrino derived information what type of elements are producing heat inside the earth, how those elements are positioned inside the earth, if those elements are moving in currents or circulations. I would also like to know how much and what type of nuclear reactions are going on inside the Sun, how much neutrino flux is coming from the interior of the earth, in relation to those produced from the sun and related to the flux produced outside the solar system. I would like to know how much nuclear reactions are going on inside the moon and what type of reactions and associated elements these are. If the design of the detector is exceptional and capable of very high resolution, I would like to know what processes are going on inside each planet and moon and why so many planets and moons are producing so much internal heat and keeping water liquid on rock solid frozen celestial bodies.

    With some imagination, such a neutrino observatory/detector might be built at a reasonable cost and with the aid of computer and/or AI support; it would answer so many questions that are impossible to answer at this current juncture.

    ReplyDelete
  25. Indeed I did not give an argument why an FCC would be worth 20B dollars. But that's because there is no algorithm for arriving at a precise figure. Was the Higgs discovery worth $10B? I tend to think so, but how would you establish that? If I was an economist I would try to determine the marginal utility of the Higgs boson vs the marginal utility of the thousands $10M experiment. Hmm.

    I think in practice the algorithm that established the worth of the LHC was something like this. There are of order 5k experimentalist at the LHC, so per experimentalist the construction cost is something like $2M. This is not so different from what an individual experimentalist could have received in terms of funding over a similar period. So I can view the LHC as researchers pooling their money, and the the real decision that was made is that physicists decided "with their feet"; they joined an LHC collaboration.

    So part of an FCC decision would be to see if these physicists (having heard your arguments) remain committed, and express interest in follow-up experiments. Now it could be that they stay committed because of inertia and groupthink and sunk cost etc. That's a plausible explanation.

    Can we prevent that? Your group think is their grit and determination. I think the only algorithm I know is that when these folks retire, physics departments have to make a decision. Yet another collider person (yawn), or a new person who builds bio-photonic nano-robots.

    ReplyDelete
    Replies
    1. Thomas,

      "Indeed I did not give an argument why an FCC would be worth 20B dollars. But that's because there is no algorithm for arriving at a precise figure."

      As I have said many times, the absolute value isn't relevant. It would be interesting to have such an estimate of course, but as you point out that is difficult if not impossible.

      What I am saying is that a next larger collider is both more expensive than other experiments in the foundations of physics and it has less motivations, hence it is not currently a good investment.

      I don't know what you are trying to achieve by dividing the costs by the number of lab members. This basically seems to be an attempt to say that particle physicists should get money for costly experiments because there are a lot of particle physicists.

      Delete
    2. Thomas,

      One serious problem with your analysis is that you don't consider the possibility that the "right" occupation for many of those HEP experimentalists is to do something else -- work in some other area of academic physics or even find employment outside of the academic world.

      This has, after all, been going on for a very long time: some of the founders of molecular biology (e.g., Max Delbrück) came out of physics, and many of us who earned Ph.D.s in high-energy-physics ended up working in other fields, ranging from the semiconductor industry, in my own case, to Wall Street.

      Physicists, after all, think of themselves as intellectually flexible.

      If steelworkers and autoworkers sometimes need to transition to new fields of endeavor, why shouldn't the same apply to physicists? Indeed, many of us can testify that it does!

      Dave

      Delete
    3. Dave,
      one of the spin-offs of particle physics research , including the accelerator labs, is exactly that trained people change field after a temporary contract and disseminate in their home countries what they learned in the inspiring atmosphere of a large research centre. .

      An accelerator lab constitutes a "critical mass" for technical training which you would not find in a university lab pursuing quantum foundations with lasers, ion traps and dilution refrigerators in a group of order of 10 persons.

      Don't misuderstand me, I do not criticise quantum foundation research, I think it is an alternative way to tackle the "big questions" for the foundations of physics, but I agree with Th. Schaeder above that one thousand experiments with an average 20 Million investment will not have the same societal, economic and technological impact as a 20 billion CHF collider facility.

      On a similar scale to accelerator-based particle physics, you can only set space research, gravitational waves and large astrophysical obsevatories. As you certainly know, all of these fields are equally criticized for their large budgets and uncertain immediate impact on the foundations of science.

      Delete
    4. I think I did try to address that. There is no algorithm for determining the right number of HEP researchers. The problem can only be crowd sourced, by individuals in HEP deciding to do something else, or by departments deciding that they are sick of hiring in HEP. Now you can argue that these mechanisms are not as efficient as they should be, but I also don't know how to determine the "right" amount of mobility between fields. Realistically, in modern science, this will almost always be small.

      Delete
    5. Sabine:

      I did make an attempt to say something about large vs small. In more detail: A possible discovery in the Higgs sector at an FCC has very large utility, but the probability is not large (let's say 10%, just to give a figure). So there is some expected utility.

      Any given small experiment (an APV experiment, for example) has discovery potential, but the utility is likely to be smaller than a Higgs sector discovery (because it will probably a discrepancy with an unidentified source, like the g-2 discrepancy), and the likelihood of discovery in any specific experiment is also smaller. The smaller expected utility can of course be overcome by performing many small experiments, and this is hard to estimate. However, I would argue that the marginal utility of many small experiments quickly declines, because you don't learn much from the 20th APV experiment. You get marginal utility only from truly independent small experiments.

      This is why I conclude that an FCC has utility so long as it does not wipe out the rest of the portfolio.

      How do we know whether the FCC crowds out small experiments? We look at the funding envelope.

      If the funding envelope is largely determined by community size, how do we know that the funding envelope is right? We don't, we can only crowd-source that decision.

      Delete
    6. Thomas Otto wrote:"one of the spin-offs of particle physics research , including the accelerator labs, is exactly that trained people change field after a temporary contract and disseminate in their home countries what they learned in the inspiring atmosphere of a large research centre. ."

      Thomas, I am one of those who actually did make the transition from HEP to industry: officially, I was a theorist, but I leaned towards phenomenology, and so had a great deal of contact with experimentalists, and even had a bit of actual hands-on experimental experience myself. I was at SLAC, so I have some feel for a big lab (I know SLAC is not nearly as big as CERN!).

      So, I really can judge whether HEP academic experience is good training for STEM in general.

      And, the answer is... it's okay. I actually figured out how to use real-space renomalization group techniques to solve a problem for chips used in the TV industry (one small part of work that helped our team win a "technical Emmy" from the Academy of Television Arts and Sciences!). But, aside from that, there was no direct transfer from my HEP training: of course, I was good at math, I could learn STEM stuff rapidly, etc.

      The problem is that, as a means of training people in STEM who end up changing fields, experimental high-energy physics is horribly, frightfully expensive. It is obviously more effective and incredibly cheaper to train people for a field they might actually work in.

      I did have a close friend with ties inside the US military-industrial complex who told me the US government funds high-energy physics mainly as a training program for physicists who will end up working on military hardware. Maybe.

      But, in terms of people who actually do productive work in civilian STEM industries, no, based on my own experience, it really does not make sense. Funding for CERN needs to stand or fall on its own, not the "spin-off" you suggest of training people who will not end up working in high-energy physics.

      Delete
  26. As a (former) experimental particle physicist, this is a difficult realization to take in. Even if I agree with you, my mind automatically struggles to find justification for building the Next Big Accelerator.

    A big part of this reaction is a psychological effect that makes humans resist large-scale change in their beliefs. The kind of change that might in some way challenge a person's sense of identity. You are probably aware of this or have even blogged about it, but I don't see anything after a quick search here.

    There are any number of articles to link to on this subject. Maybe https://www.vox.com/science-and-health/2016/12/28/14088992/brain-study-change-minds is a start. Initially, when faced with new facts that challenge one's self, the mind reacts as if experiencing pain and wants to reject it and tends to re-interpret the new facts as, impossibly, *evidence* that the new information is false! The good news, if there is any, is that change can be accomplished but it takes time and gentle repetition of the new ideas.

    Keep up the good work.

    ReplyDelete
  27. I would really stop arguing with economics. From a macroeconomic point of view, neither of the experiments is really an investment, since there is no added value or increased welfare. At least as far we can see. Money goes in one one side of CERN and goes out on the other side. It is just a redistribution and nothing is lost. The only thing here is, who gets the money to play with. Any attempt to find an economic justification in term of ROI or money spent by physicist, is rather useless and has nothing to do with what is really happening. The economics do not care, who gets the money for basic research, as long as it is spent. That is until we find something to do with this new knowledge (and here it becomes complicated and I will not start on what is a good idea to spend money on).
    I tried to show with a completely ridiculous calculation of a ROI that this does not work, but that was taken as serious. Look at it like a new ship for the Navy. A country buys it (not invests in it) and it is immediately depreciated to 1€, because it isn't an investment. There is no way, to find out an added value for the added security. Same with CERN. If the FCC is build, some companies will make a small fortune of building parts for it and then get on with real life (Spin off does not count, it is accidental).
    There is probably a higher acceptance in the population, if we spend money on CERN than on any other physics project right now and CERN is ridding that wave beautifully, but many moons ago, there where big problems that needed big experiments and CERN was born. Now it looks like, CERN wants the big experiment but is in need of a big problem to solve it with.

    ReplyDelete
  28. @sabine

    "Give me 11 million Euro and 4 years and I'll be back with a stack of proposals detailing what I recommend we do with the billions of dollars."

    Well, the millions came from EU money, Horizon2020. You should have applied for funding, which means making some rather detailed proposals of what you want to study, how, and why.
    Give it a try, Sabine: the practical life of experimenters, who need money for hardware vs that of theorists, who do not need any special hardware, except maybe for a access to high-computing environments?

    ReplyDelete
    Replies
    1. Roberto,

      Thank you, I did read the article that I linked to and I know where the funding came from. You are mistaken in claiming that I do not know how to write proposals or have never applied for funding with the EU. How about you finally stop fabricating nonsense about me?

      Delete
    2. "You are mistaken in claiming that I do not know how to write proposals or have never applied for funding with the EU. How about you finally stop fabricating nonsense about me?"

      ???
      In the lines above your comment I NEVER said that you don't know how to apply!... but clearly you didn't, because an extremely, extraordinarily bright scientist like you should have no problem convincing financing boards that their ideas are good, and deserve funding.
      Alas, it seems that writing obscure books smearing an entire scientific sector pays better...

      The EU put some money on the table, CERN and others applied for the FCC study, they got some of it... live with it.

      You stop fabricating stuff, OK? NOW! Immediately!

      Delete
  29. Sabine wrote (to Fx): You only have to convince taxpayers that that's worth $20 billion.

    I'll add that the public has to be convinced without hype and lies.

    As a member of the public, I'd like to say that I'm fed up with dismissive comments from the likes of Fx and Lorenzo.

    Fx says, "Oh come on," what I really meant is that we need to have "expert panels," but this is what he says about Sabine:

    Fx wrote: You do not understand what experimental particle physics is about.

    So, apparently, the expert panels should comprise the very experts who have a vested interest in getting funded.

    Meanwhile, Lorenzo dismisses the public with a flawed analogy:

    Lorenzo wrote: It is like if pharma companies were asking the public if one should invest in this particular molecule or another.

    No, it's not like that.

    While it's true that basic discovery research is largely funded by the public, late-stage development is largely funded by private-sector money (e.g. pharma companies and venture capitalists). It's been said that the average cost of developing a new drug is about a billion dollars.

    More importantly, the public understands how medical research is directly beneficial to them. Of course, that doesn't stop the public from complaining about the high costs of beneficial drugs, but that's another issue.

    The key point is this: If the NIH, for example, proposed a specific medical research project that cost $20 billion of public money, you can bet your bottom dollar that they would have to sell it to the public without hype and lies, and you could also bet that the public's lack of expertise would not hinder their ability to understand the potential benefits. The public could also understand if that $20 billion project failed to deliver on its promises.

    I don't feel sorry for particle physicists who have to make an honest case for new colliders. I feel sorry for climate scientists trying to convince a short-sighted public to take steps to mitigate a complex, long-term risk.

    Sabine asked Fx to explain why a new collider is a good investment for the public. We all know why it's good for particle physicists. Fx is a man of many words, but he's silent on that. Any moment now I expect Fx to trot out the "motte and bailey" argument that new colliders are not expensive on a per capita basis, and the public wastes more money on designer coffee than it would ever spend on colliders.

    Fx wrote: it would be nice if they are excited about particle physics

    Another dismissive comment. The public is interested in and excited by physics and science in general. Just about every month I see new books on science topics - indeed, too many for me to read them all. Not to mention all the monthly science magazines, in print and online, again too many to read. I might as well mention the numerous science podcasts, radio and TV programs. While I'm at it I'll mention all the extracurricular science programs for children. If Fx isn't aware of public interest in science, he's sticking his head in the sand.

    ReplyDelete
    Replies
    1. Steven,

      Perhaps I was not very clear, so let me clarify what I meant. I do care about engagement with the public, and I think outreach from physicists to explain what they are doing to the wider public is an important part of their job. It would definitely be unacceptable to ask for money for a new collider, and then do everything in secrecy. At the same time, I don't think the public should have that much of a say on whether a new collider should be built, with no offense, the public simply does not have the expertise to judge what experiments are promising and well-motivated.

      Decisions like that are taken by a panel of scientists, obviously not only comprised of particle physicists, but all of which should have some understanding of the field and of the goals at hand. I do not think Sabine should be on that committee, but that is anyway not up to me. What you say of expert panels having vested interests is conspiratorial and does not correspond to reality. These panels do not consist of CERN employees, and they do not make the final decision anyway, it goes through a whole series of steps which ends at politicians that have a strong pull towards declining and using the money for one of a thousand other things.

      As for the motivation on a new collider, I have not been silent, I have repeated several times that the primary motivation is to study the Higgs sector. Nima is saying the same thing, so are the majority of people with expertise in the field. Of course, if new physics shows up that would be even more amazing, but whether it is there or not is basically irrelevant to the case for a new collider.

      Delete
    2. Fx,

      " I have repeated several times that the primary motivation is to study the Higgs sector. Nima is saying the same thing, so are the majority of people with expertise in the field"

      Maybe or maybe not. I hope you have heard of the false consensus effect?

      Whether true or not, it is certainly not what the majority of people with expertise in the field says in public. Even if they say it, they are avoiding to make clear statements about the absence of sound predictions for BSM physics.

      Instead what the public is fed are statements like the ones in the podcast (see blogpost) or this. Your field has a problem. Do something about it.

      Delete
    3. Sabine,

      I don't deny that there are some that keep pining their hope on the premise that there are reasons to expect something around the corner. In my opinion people like Kane and Baer are somewhat delusional and are hurting the field more than anything, but okay, you can't teach an old dog new tricks and I imagine it must be difficult to accept that things didn't pan out the way they envisioned.

      I don't see any experts involved in the project doing what you are accusing them of however, which is to use BSM predictions as primary motivator for the FCC. It is still true that the FCC will allow us to probe a wide range of new models, and I don't see anything wrong with pointing that out. It is anybody's guess how likely some of these models are, but I don't think it is fair to say that all of it is useless predictions, certainly the FCC would be able to place some useful constraints at the very least. In any case, this is all secondary to the Higgs studies.

      Delete
    4. Fx,

      "people like Kane and Baer are somewhat delusional and are hurting the field more than anything, but okay, you can't teach an old dog new tricks "

      It's not Kane and Baer who "hurt the field more than anything". Every scientific discipline has some nutcases.

      This is not the problem. The problem are the thousands of people, like you, who keep their mouth shut when Baer writes for SciAM or Kane writes for Nature, because that's so convenient advertisement. One should have thought that you learned the lesson. But clearly you did not: This is not somebody else's problem. It's your problem. Do something about it.

      Delete
    5. I do not think it is convenient advertisement, what they say is out of touch and it leaves the field open to attacks such as yours. I would not mind if senior figures would correct them, but at the end of the day they are not that visible anymore and I can see the rationale for just ignoring it.

      Delete
    6. Fx,

      Then please tell me the "rationale" for just ignoring it. I have told you why it's a mistake. Physics Today reaches a lot of people in the physics community, not just particle physics. Kane's article just eroded your support base within the field.

      Your people have very publicly attacked me for writing an opinion piece whose content is flawlessly correct. I expect that you at least correct Kane's misleading statements about the discovery potential of the FCC. If you keep your mouth shut, you will further demonstrate how untrustworthy particle physicists are.

      Delete
    7. Sabine,

      I have a lot more problems with your opinion piece than with Kane's, although I disagree with the latter too. As for calling him out on his discovery claims, as I said I have no problem with that.

      But it is a bit weird to expect that of the community, he is an independent physicist with no connection to CERN, the FCC study group, or the current experiments, and he is entitled to his opinion even if it is misguided. I don't really see what attacking him would accomplish, other than giving even more attention to his article and making the HEP community look even more splintered and in-fighting. That's why I said I understand the rationale of just ignoring it. What would you want to see, an official statement from the FCC study group calling him a liar and that his is not the "party line"? What good would this do to anyone?

      I imagine your concern is about the public being mislead into believing this means the FCC will find superpartners. I sympathize with that, what can I say, it is difficult to find good sources to stay informed on science and a lot of stuff being published in popular journals is wrong or misleading. The article from Nima that you linked to would be a good place to start in my opinion.

      Delete
    8. Fx,

      I am happy to hear that my opinion piece is a problem for you.

      Sure, Kane is entitled to have an opinion, so have Bear and Dine and Lockyer and the people from CERN's PR department and so on. Those are the people who generate the hype. But the real damage comes from the rest of the community refusing to correct this hype, and excusing their behavior by saying that these are "opinions." Strange, isn't it, that those are the only opinions the public gets to hear from particle physicists?

      The article about Nima, to state the obvious, appeared in the CERN Courier which is hardly an impartial source. As a physicist (not in HEP) recently remarked to me, this magazine now looks like a sponsored ad for the FCC.

      "I imagine your concern is about the public being mislead into believing this means the FCC will find superpartners."

      No, it was my concern that the public would be mislead into believing the LHC would find superpartners, and indeed that has happened. Now my concern is that the public will conclude that particle physicists are complete idiots for thinking anyone is going to buy that story a second time.

      Delete
    9. I disagree with you on both points: the people you name (I am not sure who Lockyer is, but the others at least) are outside of the mainstream and their opinion do not represent the consensus of the HEP community. It is very easy for you to attack their arguments and make blog post after blog post about how they are lying to the public. It is not hype, it is just individual physicist who have refused to move on. It is also not true that this is the only opinions that the public gets to hear, I would say it is the opposite. In fact when I come across an article on the future of HEP, it is usually one of two: someone making reasonable arguments, like Nima, Michelangelo, Dorigo...; or an article by you that is complaining about the likes of Kane.

      I also disagree with your characterization of the CERN courier, it is a nice journal that covers a range of interesting topics in physics. They even had a rather positive review of your book a few weeks ago. In any case, my point was that if you are a layman and you want to follow the conversation on future colliders, you should listen to someone like Nima rather than Kane, whether this is in the scientific american or the cern courier is irrelevant. Sure he tends to be over the top, but at least he has a good and up to date understanding of the subject.

      Delete
    10. Fx,

      "he people you name (I am not sure who Lockyer is, but the others at least) are outside of the mainstream and their opinion do not represent the consensus of the HEP community... It is very easy for you to attack their arguments and make blog post after blog post about how they are lying to the public. It is not hype, it is just individual physicist who have refused to move on."

      You still fail to get the point. These are the people who represent your community to the public. "Hype" does not require a majority vote from the community.

      What is happening (here and elsewhere) is that a few people exaggerate, make headlines, and the rest looks away pretending they didn't notice because it's convenient advertising. When you point out the nonsense, they say "oh, but everyone knows it's nonsense". Like you do this very moment.

      Stop it.

      (Lockyer is the director of fermilab.)


      Delete
  30. It should be pointed out that with these big programs there is a corporate knowledge base that is built up. With the LHC there is a large applied science and technology corporate structure, that when ended with no follow on will dissipate. The end of the space shuttle program has a similar issue, for while that was a very expensive program (far bigger than a putative FCC) there was developed a working knowledge of how to construct and repair things in space. That may be irreversibly lost. If there is no follow on from the LHC the same may happen, unless what has been learned is taken up by the Chinese and they try to carry the torch onward.

    ReplyDelete
  31. Let's be clear "Scientists" ...

    Who cares about Truth at Microcosmic Scales??

    Is that Microcosm Truth more relevant than War issues by management of Natural Resources and COI between Social Groups that are -potentially - scalable ??

    It seems that You believe that Normal People are disable to understand the technological implications of "Knowing betting the Higgs Region" ... ( Please, do not come to Us ( normal people ) with "Scientific Obscurantisms and/or Abstract Mathematical Esoterisms" )

    People don't give a fuck about Quarks in their lifes ... Therefore, explain to the People what are the Technological consequences of "Knowing better The Higgs Region" ...

    ... Once You get those "Higgs Thresholds" .. What are the implications for improving current state-of-the-art technologies ( as Cloud networks of Quantum Computers and/or Nanotechnology and/or Satellite FMRi devices ) ???

    Most People just care about survive, pleasures,and get a life ... They - rarely - discern Truth's issues ( and less about Microcosmic aspects from Truth ) ... but People cares about Microcosmic Truths with Technological Consequences ( as Nuclear Weapons, Nuclear Plants, Microchips, Electromagnetic based communications, etc ... )

    People are not Idiots disable to understand "Why to obtain Knowledge for the Sake of Knowledge is a High Ethical Value" ... Because, That Knowledge implies Truth, ... and Truth always shows optimal and empirical facts/consequences ...

    Then, "Scientists", It is time to get down from your Ivory Tower of Mathematical Obscurantisms and began to explain why do you want Our Money for?? ...

    ReplyDelete
  32. Fx wrote: the public simply does not have the expertise to judge what experiments are promising

    And Sabine (and other physicists) don't know enough either, right?

    Fx, you are providing an excellent example of motte and bailey. You want money from the public, but the public is too ignorant even to understand any benefit to them from new colliders. That's the motte. And when a physicist like Sabine challenges the presumption that a new collider is a no-brainer project, you claim that she doesn't understand what particle physics is about. That's the bailey.

    Indeed, even with your point about the general public's lack of expertise, there's a motte and bailey. It's true that the public lacks expertise in particle physics, but that's irrelevant, just as Lorenzo's point about the public's lack of expertise in molecular biochemistry is irrelevant to drug research. The public doesn't need to be experts in aeronautics to understand the benefits of air travel, nor do they need to be experts in nuclear physics to understand the benefits of fusion power. The motte argument is the public's lack of expertise, and the bailey argument is your (and Lorenzo's) assertion that the public is too dumb to understand tangible benefits.

    Fx wrote: Perhaps I was not very clear, so let me clarify what I meant.

    You were clear before and you're clear now. We clearly see how the pro-collider community attacks anyone - including fellow physicists - who expose hype, ask questions, and challenge the status quo. Your default position was to admit that you don't care about the public and to claim that Sabine doesn't know what she's talking about. Then you try to backpedal by claiming that's not what you meant or you "weren't very clear."

    Fx wrote: As for the motivation on a new collider, I have not been silent, I have repeated several times that the primary motivation is to study the Higgs sector.

    Motte and bailey again. We already know that particle physicists want to study the Higgs sector, with a little help from billions of dollars of public money. Sabine has stated multiple times that she has no problem with that. The bailey is for you to explain how investing in a new collider will benefit the public, which you have been silent about. Since I'm a member of the public, apparently you think I'm too stupid to know when you're answering the wrong question.

    Fx wrote: if new physics shows up that would be even more amazing, but whether it is there or not is basically irrelevant to the case for a new collider.

    Irrelevant to WHOM, Fx? To the particle physicists who get a new collider? You're dismissing the public AGAIN. If you can make a case for how the new collider will benefit the public that's funding it, assuming that it won't give us any new physics, then do it.

    ReplyDelete
  33. Lawrence wrote: unless what has been learned is taken up by the Chinese and they try to carry the torch onward.

    "Taken up"? On the one hand you say that if we don't build new colliders, the know-how might be "irreversibly lost." On the other hand you say that the Chinese might carry the torch, working from scratch. Apparently you have far more confidence in the Chinese than in the entire free world.

    In the 1950's and 60's, it took a mere ten years for us to put people on the Moon, and that was when we were working from scratch with relatively primitive technology. Why are you wringing your hands about irreversible losses?

    If we decide that it's not our highest priority to send people to Mars, or back to the Moon, in the next 25 years, would that really be a problem? In the meantime, there would still be probes, rovers, satellites, space stations, etc. It's not as if we'll regress to the Stone Age if we don't colonize Mars within 25 years.

    Space might be the "final frontier," but we still have at least a thousand years worth of frontiers on Earth to keep us busy.

    ReplyDelete
  34. Fx wrote: I have a lot more problems with your opinion piece

    Then why don't you explain what the specific problems are? Please quote Sabine and tell us what's wrong. Also, I find it strange that you say that Kane is "somewhat delusional," and yet you have a lot more problems with Sabine than with Kane. Are you suggesting that Sabine is delusional?

    Let me guess: You'll tell me that you need to clarify what you meant, again. :-)

    ReplyDelete
  35. Just to make it clear, Sabine had "hinted" (the usual way she does it) at money un-deservedly gotten by the FCC design study group from EU sources.
    Before the casual reader, who could be prone to believing everything ourour nouvelle Jean d'Arc says to be true, here is a link to the totality of EU-funded Horizon2020 programs:

    https://ec.europa.eu/programmes/horizon2020/en/h2020-sections-projects

    Make your own judgement.

    Cheers.

    ReplyDelete
    Replies
    1. Roberto,

      I did not say and not "hint" at the FCC study group getting any money undeservedly. You have once again fabricated things I did not say.

      Delete
  36. For those of you who missed the earlier comments, Roberto Kersevan is an Applied Physicist at CERN's technology department. He is here to demonstrate his amazing argumentative skills that can be summed up with inventing things I didn't say.

    He previously told us that the FCC would "study and discover the origin of the universe" but now does not want to be reminded of this clearly wrong statement. He also has tried to tell my readers that particle physicists have "logically sound" reasons for why the FCC should see physics beyond the standard model, but unfortunately is not able to name any such reason.

    ReplyDelete
  37. Roberto Kersevan9:25 AM, March 15, 2019
    "You stop fabricating stuff, OK? NOW! Immediately!"

    Stay cool, man!

    ReplyDelete
  38. Kersevan wrote: our nouvelle Jean d'Arc

    I suppose that makes you our nouvelle priest trying to burn Jeanne at the stake of ad hominems.

    weristdas wrote: Stay cool, man!

    That is cool for Kersevan. I assume he acts like an adult at CERN, but in here he can be downright juvenile. Sort of a Dr. Jekyll, Mr. Hyde thing.

    ReplyDelete
  39. Sabine, I know it must be frustrating, but those of us who can follow a logical argument know that you are winning. Hang in there. It must get tedious having to argue the same point over and over again. If you played chess the way they argue, you'd be slaughtered in 15 moves.

    ReplyDelete

COMMENTS ON THIS BLOG ARE PERMANENTLY CLOSED. You can join the discussion on Patreon.

Note: Only a member of this blog may post a comment.